Is a Complete Subspace Necessarily Closed in a Metric Space?

Click For Summary
In a metric space, a complete subspace E is closed if every limit point of E is contained within E. The discussion outlines the proof that if a sequence in E converges in the larger space M, then the limit must also be in E, confirming its closed nature. Conversely, if M is complete and E is closed, then E must also be complete, as any Cauchy sequence in E converges to a point in E. The participants clarify the necessity of certain conditions in their proofs and discuss the implications of using specific values for epsilon in their arguments. Overall, the conversation emphasizes the relationship between completeness and closedness in metric spaces.
Incand
Messages
334
Reaction score
47

Homework Statement


Let ##E## be a metric subspace to ##M##. Show that ##E## is closed in ##M## if ##E## is complete. Show the converse if ##M## is complete.

Homework Equations


A set ##E## is closed if every limit point is part of ##E##.
We denote the set of all limit points ##E'##.

A point in ##p\in M## is a limit point to ##E\subseteq M## if ##\forall \epsilon > 0## ##\exists q \in E \cap B(p,\epsilon)##

The Attempt at a Solution


We want to show that ##E' \subseteq E##. Take ##p \in E'## then clearly we can choose
##p_n \in B(p,1/n)## so that ##p_n \in E##.
But then for all ##\epsilon > 0## ##d(p_n,p_m)\le d(p_n,p)+ d(p_m,p)<\epsilon## for ##n,m \ge N_\epsilon = 2/\epsilon## i.e. ##(p_n)## is a cauchy sequence.

But then it must converge to some ##\lambda \in E##. However ##\lambda = p## since ##d(p,\lambda) = d(p_n,p)+d(p_n,\lambda)< \epsilon##

Is this a correct proof? Should I use a similar approach to the second part?
 
Physics news on Phys.org
Incand said:
But then for all ##\epsilon > 0## ##d(p_n,p_m)\le d(p_n,p)+ d(p_m,p)<\epsilon## for ##n,m \ge N_\epsilon = 2/\epsilon##

Can you elaborate?
 
  • Like
Likes Incand
micromass said:
Can you elaborate?
I tried to expand the argument a bit, hopefully this works better:

We have ##d(p_n,p)<1/n## where ##p_n \in E##. By the triangle inequality we have
##d(p_n,p_m) \le d(p_n,p)+d(p_m,p) < 1/n+1/m \le 2/N_\epsilon = \epsilon## for ##n,m\ge N_\epsilon##.
So ##\forall \epsilon > 0## we have ##d(p_n,p_m) < \epsilon## for ##n,m \ge N_\epsilon = 2/\epsilon##.

Btw. I'm unclear about if using ##1/n## is neccesary at all. It seemed like a good idea at the time but perhaps starting from ##d(p_n,p)< \epsilon/2## we get ##n,m## can take any values in ##Z^+## instead,.
 
Incand said:
I tried to expand the argument a bit, hopefully this works better:

We have ##d(p_n,p)<1/n## where ##p_n \in E##. By the triangle inequality we have
##d(p_n,p_m) \le d(p_n,p)+d(p_m,p) < 1/n+1/m \le 2/N_\epsilon = \epsilon## for ##n,m\ge N_\epsilon##.
So ##\forall \epsilon > 0## we have ##d(p_n,p_m) < \epsilon## for ##n,m \ge N_\epsilon = 2/\epsilon##.

Seems ok. The only minor concern is that ##2/\varepsilon## is not necessarily an integer, so you might not be able to put it equal to ##N_\varepsilon##.

Btw. I'm unclear about if using ##1/n## is neccesary at all. It seemed like a good idea at the time but perhaps starting from ##d(p_n,p)< \epsilon/2## we get ##n,m## can take any values in ##Z^+## instead,.

I think it is definitely necessary, I don't see how you can do without.
 
  • Like
Likes Incand
So choosing ##N_\epsilon = floor(1+2/\epsilon)## should do it.

micromass said:
I think it is definitely necessary, I don't see how you can do without.
Thanks, that's good to know!Moving on to the next part I take it I'm meant to show that
##M## complete, ##E## closed ##\Longrightarrow ## ##E## complete.

Let's consider an arbitrary CS ##(p_n)## in ##E##. Since ##M## is complete it converges to a point ##p\in M##.
This means that ##\forall \epsilon > 0## ##\exists N## so that ##d(p_n,p)< \epsilon## ##\forall n \ge N##.
If ##p_n = p## for some ##n## we have ##p\in E##. If ##p_n \ne p## it means that ##p## is a limit point of ##E## since ##p_n \in E## and ##\epsilon## is arbitrary small. And since ##E' \subseteq E## since ##E## is closed we have that ##E## is complete.
 
Incand said:
So choosing ##N_\epsilon = floor(1+2/\epsilon)## should do it.


Thanks, that's good to know!Moving on to the next part I take it I'm meant to show that
##M## complete, ##E## closed ##\Longrightarrow ## ##E## complete.

Let's consider an arbitrary CS ##(p_n)## in ##E##. Since ##M## is complete it converges to a point ##p\in M##.
This means that ##\forall \epsilon > 0## ##\exists N## so that ##d(p_n,p)< \epsilon## ##\forall n \ge N##.
If ##p_n = p## for some ##n## we have ##p\in E##. If ##p_n \ne p## it means that ##p## is a limit point of ##E## since ##p_n \in E## and ##\epsilon## is arbitrary small. And since ##E' \subseteq E## since ##E## is closed we have that ##E## is complete.

Seems ok!
 
  • Like
Likes Incand
Question: A clock's minute hand has length 4 and its hour hand has length 3. What is the distance between the tips at the moment when it is increasing most rapidly?(Putnam Exam Question) Answer: Making assumption that both the hands moves at constant angular velocities, the answer is ## \sqrt{7} .## But don't you think this assumption is somewhat doubtful and wrong?

Similar threads

  • · Replies 4 ·
Replies
4
Views
2K
  • · Replies 3 ·
Replies
3
Views
1K
Replies
6
Views
2K
  • · Replies 3 ·
Replies
3
Views
2K
Replies
9
Views
3K
  • · Replies 8 ·
Replies
8
Views
3K
  • · Replies 1 ·
Replies
1
Views
1K
  • · Replies 12 ·
Replies
12
Views
4K
  • · Replies 1 ·
Replies
1
Views
2K
  • · Replies 7 ·
Replies
7
Views
2K